Solving Limit Problems with L'Hopital's Rule

In summary, the limit of the problem does not exist because the numerator does not exist and the denominator is +\infty.
  • #1
rdougie
6
0

Homework Statement



(a) Explain why L'Hopital's rule does not apply to the problem
lim[tex]_{x\rightarrow0}[/tex] [ (x[tex]^{2}[/tex]sin(1/x)) / sinx ]

(b) Find the limit.


Homework Equations



lim [tex]_{x\rightarrow0}[/tex] xsin(1/x) = 0 , by the Squeezing Theorem.

lim [tex]_{x\rightarrow0}[/tex] sin (1/x) Does Not Exist because it oscillates between -1 and 1.

lim [tex]_{x\rightarrow0}[/tex] x[tex]^{2}[/tex]sin(1/x) = 0 by the Squeezing Theorem.

lim[tex]_{x\rightarrow0}[/tex]sinx/x = 1



3. My attempt(s) at a solution

I wrote the original problem
lim[tex]_{x\rightarrow0}[/tex] [ (x[tex]^{2}[/tex]sin(1/x)) / sinx ]

as
lim[tex]_{x\rightarrow0}[/tex] sin (1/x) / lim[tex]_{x\rightarrow}[/tex](1/x) * lim[tex]_{x\rightarrow0}[/tex] (sinx/x).

Since the limit of the numerator doesn't exist, and lim[tex]_{x\rightarrow0}[/tex](1/x) is +[tex]\infty[/tex], and lim[tex]_{x\rightarrow0}[/tex] sinx/x = 1, then the limit of the problem doesn't exist, right?
 
Physics news on Phys.org
  • #2
Try thinking of the original problem as

[tex]\lim_{x\rightarrow 0} \frac{x\sin(x)}{\frac{\sin(x)}{x}}[/tex]
 
  • #3
Where did sin(1/x) go?
 
  • #4
Sorry

[tex]\lim_{x\rightarrow 0} \frac{x\sin\left(\frac{1}{x}\right)}{\frac{\sin(x)}{x}}[/tex]
 
  • #5
So now the limit is 0 if I use the Squeezing Theorem for the numerator, and the "lim sinx/x=0" for the denominator? Am I thinking about this correctly or am I just trying to plug and play?
 
  • #6
the limit is zero for the numerator, but check the list you posted in the first post again, and see what the limit for [itex]\frac{\sin(x)}{x}[/itex] is.
 
  • #7
ooh sorry lim sinx/x = 1.
ok thx!
 
  • #8
So far you haven't said anything about
(a) Explain why L'Hopital's rule does not apply to the problem
limx->0 [ (xsin(1/x)) / sinx ]
 
  • #9
rdougie said:
ooh sorry lim sinx/x = 1.
ok thx!

I think you usually use L'Hopital's rule to get that result in the first place though.

This seems like a poorly though out question to me, as any expression could be modified to make use of L'Hopitals rule, by multiplying by [itex]\frac{x}{x}[/itex], or [itex]\frac{e^{-1/x}}{e^{-1/x}}[/itex] Then taking the derivative will give you the original result.

In answer to the question a, I would say that L'hopital's rule applies, either to the expression itself as it's written, or to the denominator when you rewrite it.
 
  • #10
L'hopitals rule DOES apply (okay maybe it doesn't, but it would, if it could)! The numerator is defined on R except at 0. The denominator is defined on all of R. Both are differentiable on their respective natural domains. The derivative of the denominator is nonzero in a deleted nbd of 0. Does limit of the quotient of the derivatives exist though?

You'll want to use the sequential criterion to show that the limit does not exist.
 
Last edited:
  • #11
HallsofIvy said:
So far you haven't said anything about
(a) Explain why L'Hopital's rule does not apply to the problem
limx->0 [ (x[tex]^{2}[/tex]sin(1/x)) / sinx ]

lim x^2 = 0; lim sin(1/x) DNE; lim sinx = 0; 0 * DNE / 0 isn't one of the indeterminant forms.

thanks for the help, everbody :)
 
  • #12
hey,..

l' hopitals rule doesn't apply here because sin(1/x) and cos(1/x) oscilate rapidly at x near zero 1/x is a simple pole at x = 0,... and derivatives of all orders don't get rid of the sin(1/x) and higher derivatives give the term 1/x^n which tends to infinity as x -> 0.

at least i think so
 
  • #13
rdougie said:
lim x^2 = 0; lim sin(1/x) DNE; lim sinx = 0; 0 * DNE / 0 isn't one of the indeterminant forms.

thanks for the help, everbody :)

Not quite. x^2sin(1/x) -> 0 (product of bounded function and one going to zero) so we have 0/0 which is one of the indeterminate forms. Did you even read anything I wrote previously?
 

1. What is L'Hopital's Rule?

L'Hopital's Rule is a mathematical method used to evaluate the limit of an indeterminate form, where both the numerator and denominator approach zero or infinity.

2. When should I use L'Hopital's Rule?

L'Hopital's Rule should be used when you encounter a limit problem that cannot be solved using traditional methods, such as direct substitution or factoring.

3. How do I use L'Hopital's Rule?

To use L'Hopital's Rule, you must first rewrite the limit problem in the form of an indeterminate form, such as 0/0 or ∞/∞. Then, take the derivative of both the numerator and denominator and evaluate the limit again. Repeat this process until you are able to find a non-indeterminate form.

4. Are there any restrictions when using L'Hopital's Rule?

Yes, there are some restrictions when using L'Hopital's Rule. The limit problem must be in the form of an indeterminate form, and both the numerator and denominator must be differentiable. Additionally, the limit must be taken towards a specific value, such as approaching from the left or right.

5. Can L'Hopital's Rule be used for all types of limit problems?

No, L'Hopital's Rule can only be used for certain types of limit problems that meet the necessary requirements. There are some limit problems that cannot be solved using L'Hopital's Rule and may require alternative methods or techniques.

Similar threads

  • Calculus and Beyond Homework Help
Replies
17
Views
619
  • Calculus and Beyond Homework Help
Replies
8
Views
665
  • Calculus and Beyond Homework Help
Replies
14
Views
1K
  • Calculus and Beyond Homework Help
Replies
3
Views
1K
  • Calculus and Beyond Homework Help
Replies
2
Views
711
  • Calculus and Beyond Homework Help
Replies
10
Views
827
  • Calculus and Beyond Homework Help
Replies
6
Views
1K
  • Calculus and Beyond Homework Help
Replies
12
Views
785
  • Calculus and Beyond Homework Help
Replies
8
Views
920
  • Calculus and Beyond Homework Help
Replies
8
Views
1K
Back
Top